Đến nội dung

Tran Nguyen Lan 1107 nội dung

Có 120 mục bởi Tran Nguyen Lan 1107 (Tìm giới hạn từ 29-04-2020)



Sắp theo                Sắp xếp  

#487068 Hai đội thi đấu cờ với nhau mỗi đối thủ của đội này phải đấu với một ván với...

Đã gửi bởi Tran Nguyen Lan 1107 on 15-03-2014 - 22:36 trong Đại số

Hai đội thi đấu cờ với nhau .Mỗi đối thủ của đội này phải đấu với một ván với đối thủ của đội kia. Biết rằng tổng số ván cờ đã đấu bằng 4 lần tổng số đối thủ của 2 đội và biết rằng số đối thủ của ít nhất 1 trong 2 đội là số lẻ.Hỏi mổi đội có bao nhiêu đối thủ?

Đề đúng thế này phải ko bạn

Gọi số đối thủ đội 1 là x,đội 2 là y (người)

Ta có 1 người đội 1 sẽ đánh y ván với tất cả đối thủ đội 2

nên số ván đấu sẽ là xy (ván)

Ta có xy=4(x+y)

<=> (x-4)(y-4)=16

Mà do số đấu thủ 1 trong 2 đội là số lẻ nên 

ko mất tính tổng quát giả sử y lẻ rồi giải phương trình nghiệ nguyên là ra ngay




#486883 Trận 5 - toán rời rạc

Đã gửi bởi Tran Nguyen Lan 1107 on 14-03-2014 - 22:57 trong Thi giải toán Marathon cấp THCS 2014

Cho bàn cờ vua $8 \times 8$. Theo thứ tự từ trái qua phải, từ trên xuống dưới, ta làm việc sau:

Trong ô cờ thứ nhất đặt 1 hạt ngô

Trong ô cờ thứ hai đặt 2 hạt ngô

Trong ô cờ thứ ba đặt 4 hạt ngô

...

Trong ô cờ thứ 64 đặt $2^{63}$ hạt ngô.

 

Một con mã ô đầu tiên của bàn cờ, nó đi lòng vòng và ăn các hạt ngô trong ô nó nhảy đến( con mã di chuyển theo hình chữ L - 3 ô như đối với môn cờ vua) nhưng nó không ăn ở ô đầu tiên và không nhảy trở lại ô đầu tiên. Sau mỗi lần nó ăn người ta lại đặt số ngô bằng số ngô ban đầu vào trong ô đó. Sau khi con mã đi xong nó quay trở về ô đầu tiên và ăn nốt hạt ngô ở ô đó.

 

Hãy CM rằng số ngô mà con mã ăn chia hết cho 3.

Ta tô màu trắng đen cho bàn cờ giống như trong cờ vua, giả sử ô thứ nhất màu đen thì ta luôn có ô máu đen có số hạt dạng $2^{2k}$$\equiv 1(mod 3)$ với $k\in \mathbb{N}$, ô màu trắng có số hạt dạng $2^{2k+1}$$\equiv 2(mod 3)$ ($k\in \mathbb{N}$)

Ta dễ thấy khi đi 1 nước, con mã đi tới 1 ô khác màu ô nó đang đứng 

Vậy nên để đi từ ô thứ nhất để đi lòng vòng về ô thứ nhất ta cần có 2n nước đi ($n\in \mathbb{N}$) trong đó có n nước đi vào ô trắng , n nước đi vào ô đen 

Gọi $S_{1}$ là số hạt con mã ăn được ở trong những ô trắng thì $S_{1}\equiv 2n(mod 3)$

     $S_{2}$ là số hạt con mã ăn ở trong những ô đen, do ô thứ nhất chỉ tính là ăn 1 lần nên $S_{2}\equiv n(mod 3)$

Suy ra tổng số hạt con mã ăn được là $S=S_{1}+S_{2}\equiv 2n+n\equiv 3n(mod 3) hay S\vdots 3$ (ĐPCM) 




#486596 Kỳ Thi Chọn Học Thi Giỏi Lớp 9 Năm Học 2013-2014 tỉnh Nghệ An-môn Toán bảng A

Đã gửi bởi Tran Nguyen Lan 1107 on 13-03-2014 - 13:28 trong Tài liệu - Đề thi

Sở GD&ĐT Nghệ An                      Kỳ Thi Chọn Học Thi Giỏi Lớp 9
                                                               Năm Học 2013-2014
   Đề Chính Thức                                  Môn Thi: Toán- Bảng A
                                                    Thời gian:150 phút (không kể thời gian giao đề)
 
Câu 5 Cho đường gấp khúc khép kín có độ dài bằng $1$.Chứng minh rằng luôn tồn tại một hình tròn có bán kính $R=\dfrac{1}{4}$ chứa toàn bộ đường gấp khúc đó 

Trên đường gấp khúc lấy 2 điểm A,B sao cho A,B chia đường gấp khúc thành 2 phần = nhau =0,5
Suy ra $AB\leq 0,5$. Gọi trung điểm AB là O
Xét C là 1 điểm thuộc đường gấp khúc thì ta có $CM\leq (\frac{CA+CB}{2})\leq \frac{\frac{1}{2}}{2}=\frac{1}{4}$ (do C thuộc đường gấp khúc có độ dài =0,5
Tương tự thì khoảng cách từ O tới các điểm khác của đường gấp khúc cũng <$\frac{1}{4}$
Vậy $(O;\frac{1}{4})$ chính là đường tròn cần tìm

P/s: Đây chính là bài 203 trong sách Các bài toán hình học tổ hợp của Vũ Hữu Bình




#485185 Trận 4 - Bất đẳng thức

Đã gửi bởi Tran Nguyen Lan 1107 on 28-02-2014 - 21:15 trong Thi giải toán Marathon cấp THCS 2014

Cho $x,y\in \mathbb{R}$ thỏa mãn $(x+y)^{3}+4xy\geq 2$ . Tìm giá trị nhỏ nhất của:

$$ P=3(x^{4}+y^{4}+x^{2}y^{2})-2(x^{2}+y^{2})+1$$

Đề của 

nk0kckungtjnh

Ta có$2\leq (x+y)^{3}+4xy\leq (x+y)^{3}+(x+y)^{2}$

Suy ra $(x+y)^{3}+(x+y)^{2}-2\geq 0<=> (x+y-1)[(x+y)^{2}+2(x+y)+2]\geq 0<=> x+y\geq 1$ do$(x+y)^{2}+2(x+y)+2>0$

Ta có P=$3(x^{4}+y^{4}+x^{2}y^{2})-2(x^{2}+y^{2})+1=\frac{6(x^{4}+y^{4}+x^{2}y^{2})-4(x^{2}+y^{2})+2}{2}$

=$\frac{2(x^{4}+y^{4})-2x^{2}y^{2}+4(x^{4}+y^{4}+2x^{2}y^{2})-4(x^{2}+y^{2})+2}{2}$

=$\frac{2(x^{4}+y^{4})-2x^{2}y^{2}+4(x^{4}+y^{4}+2x^{2}y^{2})-4(x^{2}+y^{2})+2}{2}$

 =$\frac{x^{4}+y^{4}+(x^{2}-y^{2})^{2}+[2(x^{2}+y^{2})-1]^{2}+1}{2}$

Áp dụng bất đẳng thức $2(a^{2}+b^{2})\geq (a+b)^{2}$ ta có 

$x^{4}+y^{4}\geq \frac{(x^{2}+y^{2})^{2}}{2}\geq \frac{(x+y)^{4}}{8}\geq \frac{1}{8}$

Do $(x^{2}-y^{2})^{2}\geq 0,[2(x^{2}+y^{2})-1]^{2}\geq 0$

Suy ra $P\geq \frac{\frac{1}{8}+1}{2}=\frac{9}{16}$

Min P=$\frac{9}{16}$<=>x=y=$\frac{1}{2}$

 

Điểm 10 .




#484764 Chứng minh rằng $(a_{1}+a_{2})(c_{1}+c_...

Đã gửi bởi Tran Nguyen Lan 1107 on 25-02-2014 - 17:21 trong Bất đẳng thức và cực trị

Cho $a_{1},a_{2}>0$$a_{1}c_{1}\geq b_{1}^{2}$ $, a_{2}c_{2}\geq b_{2}^{2}$. Chứng minh rằng:

$(a_{1}+a_{2})(c_{1}+c_{2})\geq (b_{1}+b_{2})^2$

Ta có $c_{1},c_{2}\geq 0$

Suy ra $a_{1}a_{2}c_{1}c_{2}\geq (b_{1}b_{2})^{2}$

Mà $a_{1}c_{2}+a_{2}c_{1}\geq 2\sqrt{a_{1}c_{1}a_{2}c_{2}}\geq 2b_{1}b_{2}$

Suy ra $a_{1}c_{1}+a_{2}c_{2}+ a_{1}c_{2}+a_{2}c_{1}\geq a_{1}c_{1}+a_{2}c_{2}+2\sqrt{a_{1}c_{1}a_{2}c_{2}}\geq b_{1}^{2}+2b_{1}b_{2}b_{2}^{2}=(b_{1}+b_{2})^{2}$ (dpcm)




#484714 Cho a,b,c là 3 số thực dương thỏa mãn ab+bc+ca=3

Đã gửi bởi Tran Nguyen Lan 1107 on 24-02-2014 - 23:04 trong Bất đẳng thức và cực trị

từ $ab+bc+ca=3\Rightarrow abc\leq 1$

áp dụng ta có: $\sum \frac{1}{1+a^2(b+c)}\leq\sum \frac{1}{a(ab+bc+ca)}=\frac{1}{ab+bc+ca}.\left ( \sum \frac{1}{a} \right )=\frac{1}{abc}. "="\Leftrightarrow a=b=c=1$

Ngược dấu kìa bạn




#484212 CM: tam giác AHM vuông

Đã gửi bởi Tran Nguyen Lan 1107 on 22-02-2014 - 21:30 trong Hình học

Cho hình vuông abcd, hai điểm M, N bất kỳ với CM = CN, N nằm trên CD, M nằm trên BC. Kẻ CH vuông góc với BN tại H. CM: tam giác AHM vuông

Ta có $\angle HAB=\angle HMC=180^{\circ}-\angle BMH$

$\angle ABH=\angle MCH=90^{\circ}-\angle HBC$

=>$\angle AHB=\angle MHC$ =>$\angle AHB+\angle BHM=\angle MHC+\angle BHM<=> \angle AHM=\angle BHC=90^{\circ}$

suy ra AHM vuôing ở H




#484154 $A=5x+3y+\frac{16}{y}+\frac{12}...

Đã gửi bởi Tran Nguyen Lan 1107 on 22-02-2014 - 18:11 trong Bất đẳng thức và cực trị

Cho x, y > 0 và $x+y\geq 6$. Tìm Min:

$A=5x+3y+\frac{16}{y}+\frac{12}{x}$

Tách thành A=$2x+2y+3x+\frac{12}{x}+y+\frac{16}{y}\geq 2.6+2\sqrt{3x.\frac{12}{x}}+2\sqrt{y.\frac{16}{y}}=32$

Dấu = <=> x=2,y=4




#483638 Trận 3 - Hình học

Đã gửi bởi Tran Nguyen Lan 1107 on 17-02-2014 - 14:49 trong Thi giải toán Marathon cấp THCS 2014

MÌnh nghĩ Menelaus là 3 điểm thẳng hàng chứ sao lại là đồng quy là của Ceva mà??

Minh nham, xem bai ben duoi




#483313 Cho $a,b,c>0$ và $a+b+c=3$. CMR: $\sum...

Đã gửi bởi Tran Nguyen Lan 1107 on 15-02-2014 - 21:25 trong Bất đẳng thức và cực trị

Cho $a,b,c>0$ và $a+b+c=3$. CMR:

$\sum \frac{a^2+9}{2a^2+(b+c)^2}\leq 5$

Ta có$\frac{a^{2}+9}{2a^{2}+(b+c)^{2}}=\frac{a^{2}+(a+b+c)^{2}}{2a^{2}+(b+c)^{2}}=\frac{2a^{2}+(b+c)^{2}+2ab+2ac}{2a^{2}+(3-a)^{2}}=1+\frac{2a(b+c)}{3(a^{2}-2a+3)}$

Hay $\sum \frac{a^{2}+9}{2a^{2}+(b+c)^{2}}=3+\sum \frac{2a(b+c)}{3(a^{2}-2a+3)}\leq 5$

<=>$\sum \frac{ab+ac}{a^{2}-2a+3}\leq 3$

Mà $\frac{ab+ac}{a^{2}-2a+3}=\frac{ab+ac}{(a-1)^{2}+2}\leq \frac{ab+ac}{2}$

=$\sum \frac{ab+ac}{a^{2}-2a+3} \leq ab+ac+bc\leq 3$ suy ra DPCM




#483200 Trận 3 - Hình học

Đã gửi bởi Tran Nguyen Lan 1107 on 15-02-2014 - 11:34 trong Thi giải toán Marathon cấp THCS 2014

Do đề của các Toán thủ nộp chưa phù hợp nên trận này BTC sẽ ra đề

Đề của BTC:
Cho tam giác $ABC$ nhọn. $D,E,F$ trên $BC, CA, AB$ sao cho $\triangle DEF$ nhọn và $AD, BE, CF$ đồng quy. $M, N, P$ trên $EF, FD, DE$ sao cho $\triangle MNP$ nhọn và $DM, EN, FP$ đồng quy.

Chứng minh rằng: $AM, BN, CP$ cũng đồng quy.

Thời gian làm bài tính từ: 20h15 ngày 14/2/2014

P/s: mong các toán thủ đừng mải đi chơi với gấu mà quên làm bài :adore:

Bài trước của em dùng nhầm định lí nên xin làm lại ạ

Áp dụng định li Ceva vào tam giác ABC,DEF ta có

$\frac{DB}{DC}.\frac{EC}{EA}.\frac{FA}{FB}=1$ (*)

$\frac{ME}{MF}.\frac{NF}{ND}.\frac{PD}{PE}=1$ (**)

Kéo dài AM cắt BC ở K,BN cắt AC ở I,CP cắt AB ở H

Kẻ BB',CC',FF',EE' vuông góc với AK

Ta có$\frac{BB'}{CC'}=\frac{BK}{CK}$ (Talet)

$\frac{FF'}{EE'}=\frac{MF}{ME}$ (Talet)

$\frac{BB'}{FF'}=\frac{AB}{AF}$ (Talet)

$\frac{CC'}{EE'}=\frac{AC}{AE}$ (Talet)

Từ đây suy ra $\frac{KB}{KC}=\frac{MF}{ME}.\frac{AB}{AC}.\frac{AE}{AF}$

Tương tự $\frac{IC}{IA}=\frac{ND}{NF}.\frac{BC}{BA}.\frac{BF}{BD}$

$\frac{HA}{HB}=\frac{PE}{PD}.\frac{CA}{CB}.\frac{CD}{CE}$

Hay $frac{KB}{KC}.\frac{IC}{IA}.\frac{HA}{HB}=\frac{MF}{ME}.\frac{ND}{NF}.\frac{PE}{PD}.\frac{AE}{AF}.\frac{BF}{BD}.\frac{CD}{CE}$=1 (theo (*) và (**))

Áp dụng định lí Ceva đảo suy ra AK,BI,CH thẳng hàng hay AM,BN,CP thẳng hàng

 

Cách làm tốt

$d=10$

$S=44.5$




#483145 Trận 3 - Hình học

Đã gửi bởi Tran Nguyen Lan 1107 on 14-02-2014 - 21:56 trong Thi giải toán Marathon cấp THCS 2014

Do đề của các Toán thủ nộp chưa phù hợp nên trận này BTC sẽ ra đề

Đề của BTC:
Cho tam giác $ABC$ nhọn. $D,E,F$ trên $BC, CA, AB$ sao cho $\triangle DEF$ nhọn và $AD, BE, CF$ đồng quy. $M, N, P$ trên $EF, FD, DE$ sao cho $\triangle MNP$ nhọn và $DM, EN, FP$ đồng quy.

Chứng minh rằng: $AM, BN, CP$ cũng đồng quy.

Thời gian làm bài tính từ: 20h15 ngày 14/2/2014

 

Em không vẽ được hình,mong btc thông cảm

Áp dụng định lí Mênlauyt vào 2 tam giác ABC,DEF ta có

$\frac{DB}{DC}.\frac{EC}{EA}.\frac{FA}{FB}=1$ (*)

$\frac{ME}{MF}.\frac{NF}{ND}.\frac{PD}{PE}=1$ (**)

Kéo dài AM cắt BC ở K,BN cắt AC ở I,CP cắt AB ở H

Kẻ các đường cao BB',CC',EE',FF' xuống AK

Ta có $\frac{BK}{CK}=\frac{BB'}{CC"}$

$\frac{FF'}{EE'}=\frac{FM}{EM}$

$\frac{BB'}{FF'}=\frac{AB}{AF}$ =>$BB'=\frac{AB.FF'}{AF}$

$\frac{CC'}{EE'}=\frac{AC}{AE}$=>$CC'=\frac{AC.EE'}{AE}$

Từ đây suy ra $\frac{BK}{CK}=\frac{FM.AB.AF}{EM.AC.AE}$ (1)

Tương tự $\frac{CI}{AI}=\frac{ND.BC.BF}{NF.BA.BD}$ (2)

$\frac{AH}{BH}=\frac{PE.CA.CE}{PD.CB.CD}$ (3)

Nhân (1),(2),(3) vế theo vế

$\frac{KB}{KC}.\frac{IC}{IA}.\frac{HA}{HB}=\frac{FM.AB.AF}{EM.AC.AE}.\frac{DN.BC.BF}{FN.BA.BD}.\frac{PE.CA.CE}{PD.CB.CD}$

                                                                      =$\frac{KB}{KC}.\frac{IC}{IA}.\frac{HA}{HB}=\frac{FM.ND.PE}{ME.NF.PD}.\frac{AE.BF.CE}{AF.BD.CD}$

Theo (*) và (**) ta có$\frac{KB}{KC}.\frac{IC}{IA}.\frac{HA}{HB}=1

Áp dụng định lí Menlauyt đảo ta có AK, BI, CH đồng quy hay AM,BN,CP đồng quy

 

d=9

S=44




#478966 Giải hệ pt ẩn với ẩn x: $x^2 + y^2 = 4$ và $x^2 + (5y+2)x +4y...

Đã gửi bởi Tran Nguyen Lan 1107 on 25-01-2014 - 14:52 trong Đại số

3.Giải hê pt 4 ẩn sau:

$\left\{\begin{matrix} x+y+z+t=22 \\ xyzt=648 \\ \dfrac{1}{x}+ \dfrac{1}{y} = \dfrac{7}{12} \\ \dfrac{1}{z}+ \dfrac{1}{t} = \dfrac{5}{18} \end{matrix}\right.$

Mod sưa giúp em cái text với ạ :( Em k làm đc :((

 

Đặt x+y=a,xy=b,z+t=c,zt=d

Ta có a+c=22

          bd=648

          $\frac{a}{b}=\frac{7}{12}$=>12a=7b

         $\frac{c}{d}=\frac{5}{18}$=> 18c=5d

Suy ra 12a.18c=35bd

     <=> ac=105,kết hợp a+c=22

=> a=7,c=15 hoặc a=15,c=7

Với a=7,c=15 =>b=12,d=54

Giải ra x=3,y=4 hoặc x=4,y=3

           z=6,t=9 hoặc z=9,t=6

Tương tự cho TH còn lại




#478927 Trận 2 - PT, HPT

Đã gửi bởi Tran Nguyen Lan 1107 on 25-01-2014 - 11:30 trong Thi giải toán Marathon cấp THCS 2014

Rất xin lỗi các toán thủ đã vì post đề chậm trễ, sau đây là đề thi trận 2 MSS:

Đề của toán thủ : Best Friend

 

$$\left\{\begin{matrix} 8x^{2}+12y^{2}-20xy=0 (1)& & \\ 4x^{2}-6x+1=y^{2}-3y (2) & & \end{matrix}\right.$$

Thời gian làm bài tính từ: 23h ngày 24/1/2014

Từ (1) =>(x-y)(2x-3y)=0<=> x=y họăc 2x=3y

Với x=y thế vào (2) ta có$4x^{2}-6x+1=x^{2}-3x<=>3x^{2}-3x+1=0$

Phương trình này vô nghiệm

Với 2x=3y thế vào (2) ta có $(3y)^{2}-3.3y+1=y^{2}-3y<=> 8y^{2}-6y+1=0<=>(2y-1)(4y-1)=0$

<=> $y=\frac{1}{2} hoặc y=\frac{1}{4}$<=> $x=\frac{3}{4} hoặc y=\frac{3}{8}$

Vậy hệ có 2 nghiệm $(\frac{3}{4},\frac{1}{2}),(\frac{3}{8},\frac{1}{4})$

______________
Hỏng $\LaTeX$, chú ý diễn đàn có chức năng xem bài trước

$S = 20$ (châm trước)




#478877 Trận 2 - PT, HPT

Đã gửi bởi Tran Nguyen Lan 1107 on 24-01-2014 - 23:45 trong Thi giải toán Marathon cấp THCS 2014

Rất xin lỗi các toán thủ đã vì post đề chậm trễ, sau đây là đề thi trận 2 MSS:

Đề của toán thủ : Best Friend

 

$$\left\{\begin{matrix} 8x^{2}+12y^{2}-20xy=0 & & \\ 4x^{2}-6x+1=y^{2}-3y & & \end{matrix}\right.$$

Thời gian làm bài tính từ: 23h ngày 24/1/2014

Hệ <=>$\left\{\begin{matrix} 2x^{2}-5xy+3y^{2}=0 & & \\ (2x)^{2}-3.2x+1=y^{2}-3y & & \end{matrix}\right.$

<=>$\left\{\begin{matrix} (2x-y)(x-y)=0 & & \\ (2x)^{2}-y^{2}+3(2x-y)+1=0 & & \end{matrix}\right.$

<=>$\left\{\begin{matrix} (2x-y)(x-y)=0 (1) & & \\ (2x-y)(2x+y-3)+1=0 (2)& & \end{matrix}\right.$

Từ (2) => 2x-y$\neq 0$ kết hợp (1) => x=y

Thế vào (2) ta có (2x-x)(2x+x+3)+1=0 <=>$3x^{2}+3x+1=0$ mà phương trình này vô nghiệm suy ra hệ vô nghiệm

$S=\O$

_________________________
$S = 0$




#478776 cho a,b là các số thực thoả: $a^{3}-3a^{2}+8a=9...

Đã gửi bởi Tran Nguyen Lan 1107 on 24-01-2014 - 16:29 trong Đại số

cho a,b là các số thực thoả: $a^{3}-3a^{2}+8a=9$ (1) và $b^{3}-6b^{2}+17b=15$ (2)

tính a+b

Từ (1)=>$a^{3}-3a^{2}+3a-1=8-5a<=>(a-1)^{3}=8-5a$

(2)=>$b^{3}-6b^{2}+12b-8=7-5b$<=>$(b-2)^{3}=7-5b$

Đặt a-1=x,b-2=y=>$x^{3}=3-5x$,$y^{3}=-3-5y$

Cộng lại ta có $x^{3}+y^{3}=-5(x+y)$$x^{3}+y^{3}=-5(x+y)$

<=>$(x+y)(x^{2}-xy+y^{2}+5)=0$<=> x+y=0<=> a+b=3




#478767 Tìm m để phương trình $$(m-4)x^2 - 2(m-2)x + m - 1 = 0 (1)$...

Đã gửi bởi Tran Nguyen Lan 1107 on 24-01-2014 - 16:13 trong Đại số

Tìm m để phương trình $$(m-4)x^2 - 2(m-2)x + m - 1 = 0 (1)$$ có : 

a) Ít nhất 1 nghiệm dương

b) Đúng một nghiệm âm

c) Hai nghiệm âm.

Ta có$\Delta' =(m-2)^{2}-(m-4)(m-1)=m^{2}-4m+4-m^{2}+5m-4=m$$\geq 0$ 

Suy ra 2 nghiệm $x_{1}=m-2+\sqrt{m}$

                            $x_{2}=m-2-\sqrt{m}$

Nhận thấy $x_{2}\leq x_{1}$

a, Phương trình có ít nhất 1 nghiệm dương =>$x_{1}>0$

<=>$m+\sqrt{m}-2> 0<=> (\sqrt{m}-1)(\sqrt{m}+2)> 0<=> \sqrt{m}-1>0$ hay m> 1

b,Phương trình có đúng 1 nghiệm âm<=>$x_{1}>0>x_{2}$

<=>$\left\{\begin{matrix} m-2+\sqrt{m}>0 & & \\ m-2-\sqrt{m}<0 & & \end{matrix}\right.$

<=>$\left\{\begin{matrix} (\sqrt{m}-1)(\sqrt{m}+2)>0 & & \\ (\sqrt{m}-2)(\sqrt{m}+1)<0 & & \end{matrix}\right.$

<=>$1< \sqrt{m}<2<=> 1<m<4$

c,Phương trình có 2 nghệm âm <=> $x_{2}<x_{1}<0$

<=>$m-2+\sqrt{m}<0<=> (\sqrt{m}-1)(\sqrt{m}+2)<0$

<=>$\sqrt{m}<1<=>m<1$




#478475 $\left\{\begin{matrix}x+y=2xy & &...

Đã gửi bởi Tran Nguyen Lan 1107 on 22-01-2014 - 17:19 trong Phương trình - hệ phương trình - bất phương trình

Giải hệ phương trình:

$2)\left\{\begin{matrix}x+y=2xy(1) &  & \\ 1+yz=2y(2) &  & \\ 1+xz=2x (3)&  & \end{matrix}\right.$

(2)=> x+xyz=2xy=x+y <=>xyz=y

(3)=> y+xyz=2xy=x+y <=>xyz=x

Suy ra x=y=xyz khác 0

<=> yz=xz=1<=> 1+yz=1+xz=2=2x=2y

Suy ra x=y=z=1




#476767 $\left\{\begin{matrix} x=y^2+z^2\...

Đã gửi bởi Tran Nguyen Lan 1107 on 11-01-2014 - 22:54 trong Phương trình, hệ phương trình và bất phương trình

1) $\left\{\begin{matrix} x=y^2+z^2 (1)\\ y=z^2+x^2 (2)\\ z=x^2+y^2 (3) \end{matrix}\right.$

 

Dễ thấy x,y,z không âm

Xét x=y=z =>x=y=z=0 hoặc x=y=z=0,5

Xét 2 trong 3 số = nhau chẳng hạn x=y=>$z=2x^{2}$,$x=x^{2}+4x^{4}$=>x=0,x=0,5

Xét 3 số khác nhau

 =>$x-y=y^{2}-x^{2}=(y-x)(y+x)$

     Do $x-y\neq 0$ => x+y=-1 mà x,y không âm nên vô nghiệm

 Suy ra nghiệm phương trình là x=y=z=0 hoặc x=y=z=0,5




#475817 1.tìm x,y nguyên thỏa mãn $x^{3}+2x^2+3x+2=y^3$

Đã gửi bởi Tran Nguyen Lan 1107 on 06-01-2014 - 21:25 trong Phương trình, hệ phương trình và bất phương trình

 

2.giải hệ phương trình sau $\left\{\begin{matrix} x^{2}+y^{2}+xy+1=4y (1) & & \\ (x^{2}+1)(x+y-2)=y (2) & & \end{matrix}\right.$

2.Xét y=0 =>$x^{2}+1=0$ (vô lí)

=>$y\neq 0$ .Từ (1)=> $x^{2}+1=y(4-x-y)$

Thế vào (2) được:

y(4-x-y)(x+y-2)=y

<=> (x+y-4)(x+y-2)=-1

<=>$(x+y-3)^{2}-1=-1$ 

<=>x+y=3 đến đây thay vào (1) nữa là xong

Nghiệm (1;2) và (-2;5)




#475809 1.tìm x,y nguyên thỏa mãn $x^{3}+2x^2+3x+2=y^3$

Đã gửi bởi Tran Nguyen Lan 1107 on 06-01-2014 - 21:15 trong Phương trình, hệ phương trình và bất phương trình

1.tìm x,y nguyên thỏa mãn $x^{3}+2x^2+3x+2=y^3$

 

Rõ ràng $2x^{2}+3x+2$ dương

=>$y^{3}> x^{3}$ (1)

Với x=0 thì $y^{3}$=2 (không nguyên)

suy ra $x^{2}\geq 1$ hay $x^{2}-1\geq 0$

Ta có $(x+1)^{3}-y^{3}=x^{2}-1\geq 0$

<=>$(x+1)^{3}\geq y^{3}$ (2)

 Vậy $(x+1)^{3}\geq y^{3}>x^{3}$ nên $(x+1)^{3}= y^{3}$

Giải ra ta được x=1,y=2 hoặc x=-1,y=0




#475796 Cho tam giác ABC. Dựng ra phía ngoài

Đã gửi bởi Tran Nguyen Lan 1107 on 06-01-2014 - 20:48 trong Hình học

Cho tam giac ABC. Dựng ra phía ngoài tam giac ABC các tam giác vuông cân tại A là ABD và ACE. CMR:

       AH (đường cao của tam giác ABC) đi trung điểm I của đoạn thẳng DE

Kẻ DM,EN vuông góc AH

$\angle DAM=\angle ABH$

Suy ra $\triangle DMA=\triangle AHB$

=>DM=AH

Tương tự EN=AH

Xét 2$\triangle$ AMI và ENI bằng nhau nữa là xong




#475262 Tìm chữ số tận cùng $2^{2013^{2014}}$

Đã gửi bởi Tran Nguyen Lan 1107 on 04-01-2014 - 17:06 trong Số học

1. Tìm chữ số tận cùng của $2^{2013^{2014}}$

2. Cho $3^{n}-1\vdots n$ .Chứng minh $n\vdots 2$

B1: Dễ thấy $2^{2013^{2014}} chẵn

Do $2013\equiv 1(mod 4)$ =>$2013^{2014}\equiv 1(mod 4)$

Đặt $2013^{2014}$=4k+1

=>$2^{2013^{2014}}=2^{4k+1}=2.16^{k}$ 

Do $16^{k}\equiv 1(mod 5)$ =>$2^{2013^{2014}}$$\equiv 2(mod 5)$

Đặt $2^{2013^{2014}}$=5q+2 thì q chẵn

=>q=2t=>$2^{2013^{2014}}$=10t+2

Vậy chữ số tận cùng là 2




#470478 Tìm quỹ tích (tập hợp điểm)

Đã gửi bởi Tran Nguyen Lan 1107 on 12-12-2013 - 15:48 trong Hình học

Tìm tập hợp các điểm M trong tam giác ABC sao cho tổng diện tích của hai tam giác ABM. ACM bằng diện tích tam giác BCM.

Kéo dài AM cắt BC ở N,kẻ các đường cao BH,CK

$S_{AMB}+S_{AMC}=\frac{AM(BH+CK)}{2}$

$S_{CMB}\frac{MN(BH+CK)}{2}$

=>AM=MN

Suy ra M thuộc đường trung bình $\Delta ABC$




#470115 cho a, b thỏa mãn $a+\frac{1}{b}\leq 1...

Đã gửi bởi Tran Nguyen Lan 1107 on 10-12-2013 - 18:08 trong Bất đẳng thức và cực trị

Có phải là $a,b$ dương nữa không?

$A=\frac{a}{b}+\frac{b}{a}=\frac{a}{b}+\frac{b}{16a}+\frac{15b}{16a}\geq 2\sqrt{\frac{a}{b}.\frac{b}{16a}}+\frac{15}{16.\frac{a}{b}}$

$<=>A=\geq \frac{1}{2}+\frac{15}{16.\frac{a}{b}}$.Dấu "=" xảy ra $<=>4a=b$

Mà:

Theo giả thiết ta có:$\frac{1}{b}\leq 1-a<=>\frac{a}{b}\leq -a^2+a\leq \frac{1}{4}$

Dấu "=" xảy ra khi và chỉ khi <=>$a=\frac{1}{2};b=1$

$=> A\geq \frac{1}{2}+\frac{15}{16.\frac{1}{4}}=\frac{17}{4}$

Dấu "=" xảy ra <=>$a=2;b=1$

b=2 chứ